Уравнение Эйнштейна и тензор энергии-импульса скалярного поля

Возьмем взаимодействие гравитационного и скалярного реальных полей. Для действия гравитационного поля в вакууме добавляю член С м "=" г 4 Икс г л м , где

л м "=" 1 2 г мю ν мю ф ν ф В ( ф )
Итак, вариация дельта С м должен дать 1 2 г 4 Икс г дельта ( г мю ν ) Т мю ν , где Т мю ν относится к тензору энергии-импульса скалярного реального поля. Я пытался это сделать, но это приводит меня к ответу
дельта С м "=" г 4 Икс дельта ( г ) л м +
+ г 4 Икс г ( 1 2 дельта ( г мю ν ) мю ф ν ф г мю ν мю ф ν дельта ф В ( ф ) ф дельта ф ) "="
"=" 1 2 г 4 Икс г дельта ( г мю ν ) Т мю ν + г 4 Икс г ( г мю ν мю ф ν дельта ф В ( ф ) ф дельта ф ) .
Что делать со вторым интегралом? Равен ли он нулю по уравнению Эйлера-Лагранжа, или я не могу так сказать?

Просто придирка, но вы должны использовать ковариантные производные в своих действиях. Это не имеет значения для первых интегралов, но если вы хотите интегрировать по частям и получить EOM для вашего поля Клейна-Гордона, вам будет намного легче, если все ковариантно.
@JerrySchirmer: да, я согласен. Я использовал нековариантную производную только потому, что работал со скалярной функцией.

Ответы (2)

Выражение для дельта С м что вы ожидаете удержания при условии, что выполняемая вами вариация является вариацией только в отношении обратной метрики; не должно быть дельта ф условия. Другими словами; набор дельта ф "=" 0 , и вы получите желаемое выражение.

См., например, « Пространство-время и геометрия Кэрролла », стр. 164, он выполняет те же вычисления и явно отмечает

"Теперь измените это действие в отношении, чтобы не ф , а в обратную метрику..."

Вообще говоря, фактически тензор напряжений определяется как пропорциональный функциональной производной действия по обратной метрике;

Т мю ν "=" 2 1 г дельта С дельта г мю ν

Это правильный ответ.

Вариации действия необходимо выполнять относительно того поля, уравнения движения которого вы хотите получить. Вы изменились в отношении метрического поля и ф поле одновременно. То, что вы сделали, не является строго неправильным, поскольку варианты независимы, но привело вас в замешательство. На самом деле вы не знаете, как себя вести. Однако можно применить уравнение Эйлера-Лагранжа для поля ф а потом прийти к выводу (как и поставить дельта ф "=" 0 так как вы не меняетесь относительно ф ).

Спасибо за ответ. Но я думаю, что ваше второе равенство имеет неправильный знак:
г мю ν г мю ν "=" 4 дельта ( г мю ν г мю ν ) "=" 0 г мю ν дельта г мю ν "=" г мю ν дельта г мю ν .
Ты прав, извини. Я исправил это.